Metamath Blueprint : AKS (PRIMES is in P)


Theorem odrlblem

ReadyForStmt
$e |- G = ( Z/nZ ` r ) $.
$e |- B = ( |^ ` ( ( 2 logb N ) ^ 5 ) ) $.
$p |- ( N e. ( ZZ>= ` 3 ) -> E. r e. ( 1 ... B ) ( ( r gcd N ) = 1 /\ ( 2 logb N ) < ( od ` G ) ) ) $.
Theorem 4.1 from ~ https://www3.nd.edu/%7eandyp/notes/AKS.pdf

Making n have high order modulo r.

For n \geq 3, there exists some r \leq \left\lceil log_5 n \right\rceil such that gcd(n, r) = 1 and od_r(n) > log_2 n.